Keresés

Részletes keresés

t3kkk3n Creative Commons License 2021.05.08 0 0 575

" Mindkét szög a tetraéderből ered."

A másik dolog ami itt meglepő :

ez a két szög az optimális. Bármely más kombináció rosszabb eredményt ad.

 

És igen, leellenőriztem a Bell egyenlőtlenséggel is.

2 feletti értéket ad 22.5 fokos polarizátor eltérésnél, pont mint a QM.

t3kkk3n Creative Commons License 2021.05.08 0 0 574

Nem mondom, hogy ez a helyes megoldás, de nagyon tanúlságos, Na a téma lezárva.

 

t3kkk3n Creative Commons License 2021.05.07 0 0 573

A piros a QM jóslata. Mivel a valóságban a közös mérés soha nem éri el a nullát, a sárga megoldás teljesen jó.

Ami megdöbbentő, hogy a két oldali esemény teljesen független. 

 

Two events {} and {} are independent () if and only if their joint probability equals the product of their probabilities

https://en.wikipedia.org/wiki/Independence_(probability_theory)

 

P+=probability(amp1)*probability(amp2)*step;

t3kkk3n Creative Commons License 2021.05.07 0 0 572

Lejárt az idő.

A megfejtés: 


#include <stdio.h>
#include <stdlib.h>
#include <string.h>
#include <math.h>


#include <X11/Xlib.h>
#include <X11/Xatom.h>
#include <X11/keysym.h>

Display *display5;
Window win;
GC gc;

 

#define iradian (M_PI/180.0)

float cos2(float a)
{
a=cos(a);
return a*a;
}
float frnd(float n)
{
return n*(float)(rand()%10000)/10000.0;
}
void line(int x1,int y1,int x2,int y2,int col)
{
XSetForeground(display5,gc,col);
XDrawLine(display5, win, gc, x1,y1,x2,y2);
}
void pixel(int x,int y,int col)
{
XSetForeground(display5,gc,col);
XDrawPoint(display5, win, gc, x,y);
}

 

 


struct vec2
{
float x,y;

vec2() {x=0;y=0;}
};


float dot(vec2 v1,vec2 v2)
{
return v1.x*v2.x + v1.y*v2.y;
}
float length(vec2 v1)
{
return sqrt(dot(v1,v1));
}
void set_amp(vec2 *v,float phase,float ax)
{
v->x = sin(phase)*ax;
v->y = cos(phase)*ax;
};
void add_amp(vec2 *v,float phase,float ax)
{
v->x += sin(phase)*ax;
v->y += cos(phase)*ax;
};
float probability(vec2 v1)
{
return dot(v1,v1); // length(v1)^2
}
void add_polarizer(vec2 *v,float phase,float pol_scl)
{
vec2 pol,ori=*v;
float amp2=length(ori);

set_amp(&pol,phase,1.0);
float amp=dot(ori,pol); //v'=pol><pol|v>

if(amp<0.0) amp2=-amp2;// rotate original amplitude !!!!!!

v->x = pol.x*amp2*pol_scl; // +-length(ori)*pol_scl in pol direction
v->y = pol.y*amp2*pol_scl;
};


double w,w2,a,b,P=0,P2=0,P3=0,alpha,beta,
step=M_PI*2.0/5000.0,step2=M_PI*2.0/10.0,step3=M_PI*2.0/50000.0;

 


void getMSS()
{
alpha=38.9;beta=109.5;
//w=0.562; w2=1.0;// wrong without polarizers!
w=0.866; w2=0.65;//0.64 0.866*0.65=0.562

int enable_polarizers=1;

P=0;P2=0;P3=0;
for(double p=0;p<M_PI*2.0;p+=step)
{
double q,p1=p,p2=p,p3=p,a2=a,b2=b,a3=a,b3=b;
vec2 amp1,amp2,amp11,amp12,amp21,amp22;

p2=p + beta*iradian;

double rnd_angle1=alpha*iradian;
a2=a+frnd(rnd_angle1) - rnd_angle1/2.0;
a3=a+frnd(rnd_angle1) - rnd_angle1/2.0;
b2=b+frnd(rnd_angle1) - rnd_angle1/2.0;
b3=b+frnd(rnd_angle1) - rnd_angle1/2.0;

 

// a side
add_amp(&amp11,p1,w);
add_amp(&amp12,p2,w);
if(enable_polarizers)
{
add_polarizer(&amp11,a2,w2);
add_polarizer(&amp12,a3,w2);
}
amp1.x=amp11.x +amp12.x; // simple superposition
amp1.y=amp11.y +amp12.y;


// b side
add_amp(&amp21,p1,w);
add_amp(&amp22,p2,w);
if(enable_polarizers)
{
add_polarizer(&amp21,b2,w2);
add_polarizer(&amp22,b3,w2);
}
amp2.x= amp21.x +amp22.x;
amp2.y= amp21.y +amp22.y;


P+=probability(amp1)*probability(amp2)*step;// common detection (coincidence)
P2+=probability(amp1)*step;// independent measurement at 1 side
P3+=probability(amp2)*step;

}
P/=(M_PI*2.0);
P2/=(M_PI*2.0);
P3/=(M_PI*2.0);
}

int quantum_entanglement()
{
line(0,200, 550,200,0x555555);
line(0,550, 550,550,0x555555);
float scl=200.0*2.5;

double ww=22.5; b=ww*2.0*iradian;
step=M_PI*2.0/12000.0;

 

for(float x2=0;x2<360.0;x2+=0.2)
{
P=0,P2=0,P3=0;
a=iradian*x2;// !!!!!!!!!!!!!!! input!
int x=x2*2.5;

for(double p=0;p<M_PI*2.0;p+=step)
{
float vrnd=frnd(1.0);
P+=cos2(p-a)*cos2(p-b)*step;
P2+=cos2(p-a)*step;
P3+=cos2(p-b)*step;
}
P/=(M_PI*2.0);
P2/=(M_PI*2.0);
P3/=(M_PI*2.0);

pixel(x,550-(int)(P*scl),0x0000ff);//blue classic


P=cos2(a-b)*0.5;
pixel(x,550-(int)(P*scl),0xff0000);// red QM

 

pixel(x,550-(int)(0.0*scl),0x007700);
pixel(x,550-(int)(0.5*scl),0x007700);
pixel(x,550-(int)(1.0*scl),0x00aa00);

getMSS();
pixel(x,550-(int)(P2*scl),0xff5500);
pixel(x,550-(int)(P3*scl),0x0055ff);

pixel(x,550-(int)(P*scl),0xffff00);// yellow missing singlet state
}

return 0;
}


int main()
{
display5 = XOpenDisplay(0);
if(display5==0) printf("Do not use in root ! n");
win = XCreateSimpleWindow(display5, DefaultRootWindow(display5), 0,0, 1024, 650, 0,0,0);

XSelectInput(display5, win, StructureNotifyMask| ButtonPressMask| ButtonReleaseMask| KeyPressMask |PointerMotionMask);
XMapWindow(display5,win);
gc = XCreateGC(display5, win, 0, 0);
for(;;) { XEvent e; XNextEvent(display5, &e); if (e.type == MapNotify) break; }
XMapWindow(display5, win);

quantum_entanglement();

XFlush(display5);
getchar();

return 0;
}

 

 

t3kkk3n Creative Commons License 2021.05.07 0 0 571

A vektorok 2d síkjai a "foton" mozgási irányára merőlegesek.

Előzmény: t3kkk3n (570)
t3kkk3n Creative Commons License 2021.05.07 0 0 570

Úgy látom, hogy ez a topik áll a legközelebb ahhoz, amiről  írni akarok.

 

Szóval ez egy feladvány lesz azoknak, akik szeretnek számolni. Szóval kvantum entanglement / összefonódást és egy játék-modellt kell összehasonlítani. 

A kérdés: mi a közös bennük és mi az eltérés?

 

Legyen a "foton" most két 2d vektor. A két vektor alapesetben  109.4712° -ra állnak egymáshoz képest. A vektoroknak lehetséges szórása 38 fok. Mindkét szög a tetraéderből ered.

Namost a polarizátorok elforgatják ezeket a vektorokat úgy, hogy a két vektor egy vonalba fog esni (mindkettő ráfordul a polarizátor optikai tengelyére). Nyilván vagy ugyanarra fognak mutatni, vagy ellentétes irányba.

És itt az egyik lényeges részlet. Az utóbbit nevezzük "singlet state"-nek, ami detektálhatatlan. Ez kiesik a statisztikából. 

Ha két ellentétes irányba küldünk ilyen "foton"-párt, akkor 4 vektorral kell számolni. A random szórás a két oldalon független.

 

Mennyi a közös detektálás valószínűsége?

Mennyire tér el az eredmény a quantum entanglement-től?

Angelica Archangelica Creative Commons License 2010.07.29 0 0 566
Akkor tudnál nekem is küldeni ilyen számítógépes programot valamilyen formátumban?
Előzmény: Aurora11 (565)
Aurora11 Creative Commons License 2010.07.29 0 0 565

Szia Kedves Angelika!

 

Elvileg igen. Kimondottan erre találták ki. :P

Előzmény: Angelica Archangelica (564)
Angelica Archangelica Creative Commons License 2010.07.29 0 0 564
És rajzolni is lehet vele Feynman-gráfokat?
Előzmény: Aurora11 (562)
ic3fox Creative Commons License 2010.07.29 0 0 563
" fénysebességnél gyorsabb haladás csak a relativitáselmélet Einsten-féle változatában jelent „valódi”, az idõben visszafelé történõ mozgást, mert ebben az elméletben az összes Lorentz-féle vonatkoztatási rendszer egyenértékû."
http://felpakol.uw.hu/map/schrodinger_kiscicai.htm


A megoldas egyszeru...
Aurora11 Creative Commons License 2010.07.24 0 0 562
Tudom, sajnos ez így van. Idén nyáron a form nevű programot kell megtanulnom. Ezzel Feynman gráfokat lehet számolni.
Előzmény: mmormota (561)
mmormota Creative Commons License 2010.07.24 0 0 561
Valamilyen szinten előbb-utóbb úgyis rákényszerülsz. :-)
A te szakmádban nem lehet megúszni.
Előzmény: Aurora11 (560)
Aurora11 Creative Commons License 2010.07.24 0 0 560

Köszönöm szépen!

 

Kimaradt az életemből a programozás!:)

Előzmény: mmormota (559)
mmormota Creative Commons License 2010.07.24 0 0 559
C nyelv.

if(o==0) f1=a1

Ez a következőt jelenti: ha teljesül az hogy o=O akkor az f1 változó vegye fel az a1 változó értékét.
Előzmény: Aurora11 (558)
Aurora11 Creative Commons License 2010.07.24 0 0 558

Szia Index4!

 

"if(o==0) f1=a1;
if(o==1) f1=a1;
if(o==2) f1=a1+M_PI/2;
if(o==3) f1=a1+M_PI/2;

vagy
if(o==0) f1=a2;
if(o==1) f1=a2;
if(o==2) f1=a2+M_PI/2;
if(o==3) f1=a2+M_PI/2;"

 

Ne haragudj, de nem ismerem ezeket e jelöléseket. Tudsz ebben segíteni?

 

"Az időbeli visszahatás és a végtelen sebességű kapcsolat megkülönböztethetetlen.
Mivel az előbbi értelmetlen, maradjunk az utóbbinál."

 

Igen, persze. Az időbe visszafelé terjedő megoldásokat ki kell zárni. Ez a hullámokat leíró differenciálegyenletekre vonatkozó egyik határfeltétel, amit kauzalitásnak hívnak.

 


 

Előzmény: ind4x (557)
Aurora11 Creative Commons License 2010.05.05 0 0 556

Szia!

 

Van olyan, hogy idődimenzió.

 

"De jelenleg csak ezt feltetelezve lehet modellezni az EPR fotonokat ugy, hogy az egyedi polarizatoroknal teljesuljon a Malus-torveny."

 

Szerintem ez nem elfogadható modell. Mert az ellentétes időirányú folyamatokat ki kell zárni.

Előzmény: 3mmett (555)
3mmett Creative Commons License 2010.05.03 0 0 555
Idoben nem mehet visszafele semmi.
Foleg, ha nincs is olyan, hogy idodimenzio.

De jelenleg csak ezt feltetelezve lehet modellezni az EPR fotonokat ugy, hogy az egyedi polarizatoroknal teljesuljon a Malus-torveny.

Vagy van fenyerovaltozas, csak elhallgatjak. ;-)

Aurora11 Creative Commons License 2010.05.02 0 0 554
Minden jelenlegi elmélet ilyen. Mert a QED is egy fenomelogikus elmélet, mint a magfizika vagy a termodinamika. A jövőnek kell megadni azt a mikroszkopikus elméletet, ami képes lenne elméleti úton is visszaadni a természeti állandókat.
Előzmény: 3mmett (549)
Aurora11 Creative Commons License 2010.05.02 0 0 553

Szia!

 

Hogyan terjedhet egy jel időben visszafelé. Szerintem az, hogy csak a retardált megoldásokat fogadjuk el, és az avanzsált ás állóhullám megoldásokat kizárjuk az a határfeltételek egy fajtája. Ezt nem lehet megszegni! Gondold el, ha például nem zárnánk ki a húrrezgéseknél azokat a megoldásokat, amik a végeknél nem nulla kitérést írnak elő, akkor nem lenne a húrrezgés kvantált. Ugyanígy a harmonikus oszcillátorok sem lennének kvantáltak, hogyha a végtelenbe nem lecsengő hullámfüggvényeket nem zárnánk ki egyfajta határfeltétel megkövetelésével.

 

A retardált megoldásokon kívűl mind az avanzsált(időben visszafelé menő), mind az állóhullámmegoldásokat(időben előre és időben visszafelé menő megoldás számtani közepe) ki kell zárni. Ez a határfeltétel a kauzalitás.

Előzmény: 3mmett (552)
3mmett Creative Commons License 2010.05.02 0 0 552
"Ilyen értelemben lehet az Einsteni determinisztikus kvantummehanikát űzni,"

Nem lehet.

http://en.wikipedia.org/wiki/Quantum_teleportation
"Although Alice and Carol never interacted with each other, their particles are now entangled."

Hogy hathatna mar ket olyan foton egymasra, amelyek nem is tudnak egymas letezeserol?
Csak idoben visszafele terjedo jellel.
Előzmény: Aurora11 (546)
3mmett Creative Commons License 2010.05.02 0 0 551
Igy van.
A vegtelen sebessegu hatassal az a problema, hogy mindenkepp fenyerovaltozas lepne fel az ellenkezo meresi oldalon. Jelenleg az EPR parokat eleg hatekonyan lehet eloallitani ppktp kristalyokkal, ahol mar eszreveheto lenne ez a fenyerovaltozas.


Előzmény: Callie (547)
Angelica Archangelica Creative Commons License 2010.05.02 0 0 550
Igen, éppen ezért NEM azt mondjuk, hogy "nincsen antirészecskéje"......
Előzmény: 3mmett (548)
3mmett Creative Commons License 2010.05.02 0 0 549
" Gondolom ezt nem tehetné meg, ha a kvantummechanikából hiányoznának a rejtett paraméterek."

A QED pontossaga egy nagyon erdekes tema. Valojaban mindig van bemeno parameter, amit egy masik kiserletbol kell felvenni. A QED onmagaban nem adja meg ezt a pontossagot. Az kulonbozo tipusu kiserletek kozt teremt nagyon preciz kapcsolatot.
Előzmény: Aurora11 (545)
3mmett Creative Commons License 2010.05.02 0 0 548
Onmaga, mert szamunkra megkulomboztethetetlenek. De attol meg az egy antifoton.
Előzmény: Angelica Archangelica (543)
Callie Creative Commons License 2010.05.01 0 0 547

Ugye eloszor hatarozottan tagadja, hogy klasszikusan szimulalhato lenne a kvantumfizika, azutan bevallja, hogy vegtelen sebessegu "hatasnal" megiscsak lehetseges olyan klasszikus leirast kesziteni, ami visszaadja a kvantummechanika furcsasagait.

 

Az, hogy végtelen sebeséggel hatást közvetítő rejtett paraméterekkel a QM-el egyenértékű determinisztikus elméletet lehet felépíteni, trivialitás. Vannak ugyanis ilyen modellek, a leghíresebb köztük a Bohm-féle kvantumpotenciálos.

Ezek visszaadják a QM eredményeit - az kérdéses, hogy tudnak-e többet is.

 

Úgyhogy ilyesmit "tagadni" elég értelmetlen.

Természetesen a Bell-kísérletek sem állnak ellentmondásban ilyen modellel.

Előzmény: xdman (535)
Aurora11 Creative Commons License 2010.05.01 0 0 546

"Kizarolag a mult ismereteben nem josolhato meg a jovo, mert az bizonyos mertekben az visszahat. Ennek a hatarozatlansagnak valahogy koze van a kvantummechanika hatarozatlansagi tetelehez."

 

Ha nem az a feladatot, hogy az elemi részecskék pályáját r(t) határozd meg, akkor a téroperátorok megadásával teljesen jellemezheted a mikroszkopikus rendszert. Az egyes részecskék pályájának megadása, amúgy is lényegtelen információ, és a determinisztikusság kérdése alatt mindig is ezt tartották. Valójában, ha a téroperátorok Fi(r,t) megadása már teljesen jellemzi a rendszert.

 

  Ki lehet kerülni a határozatlansági relációból fakadó bizonytalanságot, ha csak egymással páronként kommutáló operátorokkal számolsz, és jellemzed a kvantumállapotokat(mert ezeket egy időben tetszőleges pontossággal lehet mérni). Ugyanis a nem kommutáló operátorok között van határozatlanság.

Ilyen értelemben lehet az Einsteni determinisztikus kvantummehanikát űzni, persze a téroperátorok időfejlődése determinisztikus(kölcsönhatás hiányában). Ilyenkor persze a determinisztikusság nem azt jelenti, hogy a kvantumok pályáját adjuk meg determinisztikusan, hanem a téroperátorok hely- és időfüggését. A téroperátorok várhatóértékei a klasszikus térerősségek.

 

A téridő dimenzióval kapcsolatban azt mondom, hogy a valós térnek a háromdimenziós (x,y,z) teret tekintsd. És a kölcsönhatás véges sebességének(fénysebesség) a mozgásokra való visszahatását úgy veheted figyelembe, ha bevezetsz (ct,x,y,z) négyesvektorokat. És a vektorok egy négydimenziós vektortér elemei, amit téridőnek hívunk. De ezt már absztrakt értelemben vedd térnek. A részecske haladási irányát x-nek véve a ct időtengelyt merőlegesen felvéve, kétdimenziós térként(síkként) ábrázolhat a téridőt, vagy pedig a mozgás síkját x- és y- koordinátával kifeszítve a ct időtengely háromdimenziós térként ábrázolható. De teljesen sohasem ábrázolható, nem érdemes az elképzeléssel törödni, ugyanis csak félrevezeti az embert. Ugyanis az ember képzelete csak euklideszi geometriát képes felfogni, Minkovszky geometria amúgy sem megy.

 

Szerintem a téridőt egyszerűen vektortérnek tekinsd, és algebrailag számolj. A téridő, mint a négyesvektorok vektortere, arra szolgál, hogy a négyesvektorok által a mozgástörvények relativisztikussága már alakilag is felismerhető legyen. A lényeg az, hogy a kölcsönhatás véges sebessége fékező hatással van a mozgásra. Ennek figyelembe vételéhez kell a relativitáselmélet.

Előzmény: 3mmett (542)
Aurora11 Creative Commons License 2010.05.01 0 0 545

Ez érdekes dolog. Elvileg szerintem a terek kommutációs reláció tartalmazzák az EPR kísérlet feloldását is. Mert a QED például az elektron mágneses momentumát 13 tizedes jegy pontossággal megjósolja. Gondolom ezt nem tehetné meg, ha a kvantummechanikából hiányoznának a rejtett paraméterek.

Biztos emlékeztek eldinből, hogy Coulomb mértékből az elektromágneses skalárpotenciál pillanatszerűen változik(végtelen sebességgel), míg a vektorpotenciál szintén hullámszerűen terjed. Ez sérti a kauzílitást. Ez nem probléma, mert nem a potenciálok fontosak fizikailag, hanem a térerősségek, amik Coulomb mértékben is hullámszerűen terjednek és nem sértik a kauzalitást.

Meg kellene vizsgálni, hogy az EPR kisérletben a megfigyelt mennyiségek nem olyan segédmennyiségek, mint a skalár vagy vektorpotenciál. Mert csak a fizikailag kimérhető mennyiségekre kell a kauzalitásnak teljesülnie. Nem-e lehetséges, hogy a spinbeállások pillanatszerű változása olyan, mint a Coulomb mértékbeli elektromágneses skalárpotenciál?

Előzmény: 3mmett (539)
Aurora11 Creative Commons License 2010.05.01 0 0 544

Szia!

 

Nem terjedhet a hatás végtelen sebességgel. Ez elég ahoz, hogy egy modell hibás legyen, mert a téridő szimmetriák abszolút nem lesznek igazak.

Előzmény: xdman (535)
Angelica Archangelica Creative Commons License 2010.05.01 0 0 543
A foton antirészecskéje önmaga, mert elektromos-, barion-, leptontöltése, ritkasága, hipertöltése, stb. egyaránt nulla, ezért részecske-antirészecske konjugáció során önmagába transzformálódik.....
Előzmény: 3mmett (541)
3mmett Creative Commons License 2010.05.01 0 0 542
Kizarolag a mult ismereteben nem josolhato meg a jovo, mert az bizonyos mertekben az visszahat. Ennek a hatarozatlansagnak valahogy koze van a kvantummechanika hatarozatlansagi tetelehez.
Tovabba ez egy eros megerositese annak, hogy az einsteini terido idodimenzioja tenyleg egy valos dimenzio, bar ezt tovabbra se tudom teljesen elfogadni. De ez az en bajom.

Ha kedveled azért, ha nem azért nyomj egy lájkot a Fórumért!